Merge remote branch 'public/master'
[course.git] / latex / problems / Serway_and_Jewett_4 / problem04.51.tex
1 \begin{problem*}{4.51}
2 If you jump from a desktop and land stiff-legged on a concrete floor,
3 you run a significant rist that you will break a leg.  To see how that
4 happens, consider the average force stopping your body when you drop
5 from rest from a height of $h = 1.00\U{m}$ and stop in a much shorter
6 distance $d$.  Your leg is likely to break at the point where the
7 cross-sectional area of the tibia is smallest.  This point is just
8 above the anke, where the cross sectional area of one bone is about $A
9 = 1.60\U{cm}^2$.  A bone will fracture when the compressive stress on
10 it exceeds about $\sigma_b = 1.60\E{8}\U{N/m}^2$.  If you land on both
11 legs, the maximum force $F_{max}$ that your ankles can safely exert on
12 the rest of your body is then about
13 \begin{equation}
14  F_{max} = 2 F_b = 2 \sigma_b A = 5.12\E{4}\U{N}
15 \end{equation}
16 Calculate the minimum stopping distance $d$ that will not result in a 
17  broken leg if your mass is $m = 60.0\U{kg}$.
18 \end{problem*} % problem 4.51
19
20 \begin{solution}
21 The problem breaks down into two constant-acceleration problems.
22 Call the top of the desk dropping-off-point $P_0$, the point of maximum 
23 velocity when you are just starting to contact the floor $P_1$, and the 
24 point where your shoe soles have compressed a distance $d$ and brought 
25 you back to rest $P_2$.
26
27 First consider the constant acceleration portion from $P_0$ to $P_1$.
28 Your final velocity $v_1$ is given by
29 \begin{equation}
30  v_1^2 = v_0^2 + 2 a_{01} \Delta y_{01}
31        = 2 a_{01} \Delta y_{01}
32 \end{equation}
33 Now applying the same equation to
34  the second constant acceleration portion from $P_1$ to $P_2$.
35 \begin{align}
36  v_2^2 &= v_1^2 + 2 a_{12} \Delta y_{12} = 0 \\
37  v_1^2 &= -2 a_{12} \Delta y_{12} = 2 a_{01} \Delta y_{01} \\
38  d &= \Delta y_{12} = -\frac{a_{01}}{a_{12}} \Delta y_{01}
39 \end{align}
40 The acceleration $a_{12}$ is given by Newton's second law
41 (picking down as the $+\vect{x}$ direction)
42 \begin{align}
43  \sum F_x &= m a_{12x} \\
44   a_{12x} &= (\sum F_x)/m
45         = \frac{mg - F_{max}}{m}
46         = g - \frac{F_{max}}{m}
47 \end{align}
48 (I forgot to include $mg$ in the sum of the forces when I was doing
49 the problem, so I didn't take off points if you forgot it as well.)
50 So
51 \begin{align}
52  d &= -\frac{a_{01}}{a_{12}} \Delta y_{01}
53     = -\frac{g}{g - \frac{F_{max}}{m}} h
54     = -\frac{1}{1 - \frac{F_{max}}{mg}} h \\
55    &= -\frac{1}{1 - \frac{5.12\E{4}}{60 \cdot 9.8}} \cdot 1.00\U{m}
56     = \ans{1.16\U{cm}}
57 \end{align}
58
59 (Ignoring gravity in your sum of forces, you would have gotten
60  $d = \frac{mg}{F_{max}} h = 1.15\U{cm}$.
61 The correction is very small because $F_{max} \gg mg$.)
62 \end{solution}